1answer.
Ask question
Login Signup
Ask question
All categories
  • English
  • Mathematics
  • Social Studies
  • Business
  • History
  • Health
  • Geography
  • Biology
  • Physics
  • Chemistry
  • Computers and Technology
  • Arts
  • World Languages
  • Spanish
  • French
  • German
  • Advanced Placement (AP)
  • SAT
  • Medicine
  • Law
  • Engineering
vivado [14]
3 years ago
7

Last one help. Use formulas to find the lateral area and surface area of the given prism. Round your answer to the nearest whole

number.
A. 645 m2; 812 m2
B. 668 m2  ; 704m2
C. 645 m2; 740 m2
D. 668 m2; 740 m2

Mathematics
1 answer:
Fed [463]3 years ago
3 0
The formula for finding the lateral area for a triangular prism is
A_{L} = (a + b + c )h

So put in the values and you get

A_{L} = (12.04 + 9 + 8 )23
A_{L} = 668

The formula for finding the surface area of a triangular prism is
SA = bh + (s1 + s2 + s3)H

So put in all the values and you get

SA = 9(8) + (9 + 8 + 12.04)23
SA = 740

Hope this helps! :)
You might be interested in
I need to know how to find the perimeter please.
shutvik [7]
First, take all the sides you already have, then add them up. (You should get 12) To find the other two sides, do some more simple addition. The 2+3+2 of the the top side add up to six (the bottom side) and sides equal each other so add another three. Total is 12+6+3=21
5 0
3 years ago
Read 2 more answers
Hello, Brainly community!
ioda

Answer:

(B)  \displaystyle \frac{W(3.1) - W(2.9)}{0.2}

General Formulas and Concepts:

<u>Calculus</u>

Limits

Derivatives

  • The definition of a derivative is the slope of the tangent line.

Derivative Notation

Instantaneous Rates

  • Tangent Line: \displaystyle f'(x) = \frac{f(b) - f(a)}{b - a}

Step-by-step explanation:

Since we are trying to find a <em>rate</em> at which W(t) changes, we must find the <em>derivative</em> at <em>t</em> = 3.

We are given 2 close answer choices that would have the same <em>numerical</em> answer but different <em>meanings</em>:

  1. (A)  \displaystyle  \lim_{t \to 3} W(t)
  2. (B)  \displaystyle \frac{W(3.1) - W(2.9)}{0.2}

If we look at answer choice (A), we see that our units would simply just be volume. It would not have the units of a rate of change. Yes, it may be the closest numerically correct answer, but it does not tell us the <em>rate</em> at which the volume would be changing and it is not a derivative.

If we look at answer choice (B), we see that our units would be cm³/s, and that is most certainly a rate of change. Answer choice (B) is also a <em>derivative</em> at <em>t</em> = 3, and a derivative tells us what <em>rate</em> something is changing.

∴ Answer choice (B) will give us the best estimate for the value of the instantaneous rate of change of W(t) when <em>t</em> = 3.

Topic: AP Calculus AB/BC (Calculus I/I + II)

Unit: Differentiation

Book: College Calculus 10e

8 0
2 years ago
A mosaic at the art museum is divided into 6 sections that each contain about the same number of tiles. There are 2,889 tiles to
AlexFokin [52]

Answer

Find out the how many tiles are in each section .

To prove

Let us assume that the number of tiles are in section are be x.

As given

A mosaic at the art museum is divided into 6 sections that each contain about the same number of tiles.

There are 2,889 tiles total in the mosiac .

Than the equation becomes  

6 × x = 2889

x = \frac{2889}{6}

x = 481.5

Therefore the total number of tiles in each sections are 481.5 .


7 0
3 years ago
Can someone help me?<br> (A) 1.5<br> (B) 3<br> (C) 4.5<br> (D) 6
zhenek [66]

Find the horizontal distance of 230 and find the Vertical distance , which is where the black dot is located.

The black dot is on 49 inches.

Now find the vertical distance f the black line at horizontal 230: This is on 47.5.

The difference between the two is : 49 - 47.5 = 1.5

The answer would be A. 1.5

6 0
3 years ago
Read 2 more answers
ASAP HELPPPPPPPPPPPPPPPP!!!!!!!!!!!!!!!
melomori [17]

Step-by-step explanation:

  1. 90-27=63°
  2. <2
  3. <2,<4
  4. 360°
3 0
2 years ago
Read 2 more answers
Other questions:
  • A closed container has 5.04 ⋅ 1023 atoms of a gas. Each atom of the gas weighs 1.67 ⋅ 10–24 grams.
    6·1 answer
  • the larger of two numbers is 3 more than the smaller number if the sum of the numbers is 73 what are the two numbers
    11·1 answer
  • How many properties of parallelograms are there?
    14·1 answer
  • If t = 4 find the value of t+6 over 5 (fraction)
    6·1 answer
  • Use the gcf to factor: 6e+24n​
    13·1 answer
  • What is the similarity?
    13·1 answer
  • 2 times the radius equals the....
    15·1 answer
  • A number cube numbered 1-6 is rolled 30 times and lands on an even number 18 times. How
    13·1 answer
  • Which vocabulary word best describes what has been circled in this expression?
    11·1 answer
  • A car cover a distance 300 miter in 50 second How much distance a car covers in 1 second​
    10·1 answer
Add answer
Login
Not registered? Fast signup
Signup
Login Signup
Ask question!